$S^{-1}Ext^n_R(A,B) cong Ext^n_{S^{-1}R}(S^{-1}A,S^{-1}B)$












1












$begingroup$


I am trying to understand a proof from the book An Introduction to Homological Algebra, Weibel.




Let $A$ be a finitely generated module over a noetherian ring $R$. Then for every multiplicative set $S$, all modules B, and all n, $S^{-1}Ext^n_R(A,B) cong Ext^n_{S^{-1}R}(S^{-1}A,S^{-1}B)$.




PROOF:



Choose a resolution $Flongrightarrow A$ by finitely generated free $R$-modules. Then $S^{-1}F longrightarrow S^{-1}A$ is a resolution by finitely generated free $S^{-1}R$-modules.



Because $S^{-1}$ is an exact functor from $R$-modules to $S^{-1}R$-modules, we have:



$S^{-1}Ext^n_R(A,B) = S^{-1}(H^* Hom_R(F,B)) cong H^*(S^{-1}Hom_R(F,B)) cong H^*Hom_{S^{-1}R}(S^{-1}F, S^{-1}B) = Ext^n_{S^{-1}R}(S^{-1}A,S^{-1}B)$



QUESTIONS:




  1. I have seen in my lectures that $Ext_R^n(M,N)$ are the right derived functor cohomologies of some module $N$ with respect to $h^M$. And you need a complex $0 longrightarrow M longrightarrow I_0 longrightarrow I_1 longrightarrow ...$, where $I_i$ are injective objects, to compute it. But here he has a resolution $.... longrightarrow F_0longrightarrow A longrightarrow 0$ (I mean, it goes the other way). Help?


  2. I do not understand why $S^{-1}(H^* Hom_R(F,B)) cong H^*(S^{-1}Hom_R(F,B))$. Help?


  3. In case that $A$ is not finitely generated, is this true? I guess it is not. But how could I prove it?


  4. Why do we need to consider the resolution $S^{-1}F longrightarrow S^{-1}A$ if we only computate $Ext^n_R(A,B)$?



Thank you very much.










share|cite|improve this question











$endgroup$

















    1












    $begingroup$


    I am trying to understand a proof from the book An Introduction to Homological Algebra, Weibel.




    Let $A$ be a finitely generated module over a noetherian ring $R$. Then for every multiplicative set $S$, all modules B, and all n, $S^{-1}Ext^n_R(A,B) cong Ext^n_{S^{-1}R}(S^{-1}A,S^{-1}B)$.




    PROOF:



    Choose a resolution $Flongrightarrow A$ by finitely generated free $R$-modules. Then $S^{-1}F longrightarrow S^{-1}A$ is a resolution by finitely generated free $S^{-1}R$-modules.



    Because $S^{-1}$ is an exact functor from $R$-modules to $S^{-1}R$-modules, we have:



    $S^{-1}Ext^n_R(A,B) = S^{-1}(H^* Hom_R(F,B)) cong H^*(S^{-1}Hom_R(F,B)) cong H^*Hom_{S^{-1}R}(S^{-1}F, S^{-1}B) = Ext^n_{S^{-1}R}(S^{-1}A,S^{-1}B)$



    QUESTIONS:




    1. I have seen in my lectures that $Ext_R^n(M,N)$ are the right derived functor cohomologies of some module $N$ with respect to $h^M$. And you need a complex $0 longrightarrow M longrightarrow I_0 longrightarrow I_1 longrightarrow ...$, where $I_i$ are injective objects, to compute it. But here he has a resolution $.... longrightarrow F_0longrightarrow A longrightarrow 0$ (I mean, it goes the other way). Help?


    2. I do not understand why $S^{-1}(H^* Hom_R(F,B)) cong H^*(S^{-1}Hom_R(F,B))$. Help?


    3. In case that $A$ is not finitely generated, is this true? I guess it is not. But how could I prove it?


    4. Why do we need to consider the resolution $S^{-1}F longrightarrow S^{-1}A$ if we only computate $Ext^n_R(A,B)$?



    Thank you very much.










    share|cite|improve this question











    $endgroup$















      1












      1








      1





      $begingroup$


      I am trying to understand a proof from the book An Introduction to Homological Algebra, Weibel.




      Let $A$ be a finitely generated module over a noetherian ring $R$. Then for every multiplicative set $S$, all modules B, and all n, $S^{-1}Ext^n_R(A,B) cong Ext^n_{S^{-1}R}(S^{-1}A,S^{-1}B)$.




      PROOF:



      Choose a resolution $Flongrightarrow A$ by finitely generated free $R$-modules. Then $S^{-1}F longrightarrow S^{-1}A$ is a resolution by finitely generated free $S^{-1}R$-modules.



      Because $S^{-1}$ is an exact functor from $R$-modules to $S^{-1}R$-modules, we have:



      $S^{-1}Ext^n_R(A,B) = S^{-1}(H^* Hom_R(F,B)) cong H^*(S^{-1}Hom_R(F,B)) cong H^*Hom_{S^{-1}R}(S^{-1}F, S^{-1}B) = Ext^n_{S^{-1}R}(S^{-1}A,S^{-1}B)$



      QUESTIONS:




      1. I have seen in my lectures that $Ext_R^n(M,N)$ are the right derived functor cohomologies of some module $N$ with respect to $h^M$. And you need a complex $0 longrightarrow M longrightarrow I_0 longrightarrow I_1 longrightarrow ...$, where $I_i$ are injective objects, to compute it. But here he has a resolution $.... longrightarrow F_0longrightarrow A longrightarrow 0$ (I mean, it goes the other way). Help?


      2. I do not understand why $S^{-1}(H^* Hom_R(F,B)) cong H^*(S^{-1}Hom_R(F,B))$. Help?


      3. In case that $A$ is not finitely generated, is this true? I guess it is not. But how could I prove it?


      4. Why do we need to consider the resolution $S^{-1}F longrightarrow S^{-1}A$ if we only computate $Ext^n_R(A,B)$?



      Thank you very much.










      share|cite|improve this question











      $endgroup$




      I am trying to understand a proof from the book An Introduction to Homological Algebra, Weibel.




      Let $A$ be a finitely generated module over a noetherian ring $R$. Then for every multiplicative set $S$, all modules B, and all n, $S^{-1}Ext^n_R(A,B) cong Ext^n_{S^{-1}R}(S^{-1}A,S^{-1}B)$.




      PROOF:



      Choose a resolution $Flongrightarrow A$ by finitely generated free $R$-modules. Then $S^{-1}F longrightarrow S^{-1}A$ is a resolution by finitely generated free $S^{-1}R$-modules.



      Because $S^{-1}$ is an exact functor from $R$-modules to $S^{-1}R$-modules, we have:



      $S^{-1}Ext^n_R(A,B) = S^{-1}(H^* Hom_R(F,B)) cong H^*(S^{-1}Hom_R(F,B)) cong H^*Hom_{S^{-1}R}(S^{-1}F, S^{-1}B) = Ext^n_{S^{-1}R}(S^{-1}A,S^{-1}B)$



      QUESTIONS:




      1. I have seen in my lectures that $Ext_R^n(M,N)$ are the right derived functor cohomologies of some module $N$ with respect to $h^M$. And you need a complex $0 longrightarrow M longrightarrow I_0 longrightarrow I_1 longrightarrow ...$, where $I_i$ are injective objects, to compute it. But here he has a resolution $.... longrightarrow F_0longrightarrow A longrightarrow 0$ (I mean, it goes the other way). Help?


      2. I do not understand why $S^{-1}(H^* Hom_R(F,B)) cong H^*(S^{-1}Hom_R(F,B))$. Help?


      3. In case that $A$ is not finitely generated, is this true? I guess it is not. But how could I prove it?


      4. Why do we need to consider the resolution $S^{-1}F longrightarrow S^{-1}A$ if we only computate $Ext^n_R(A,B)$?



      Thank you very much.







      abstract-algebra commutative-algebra proof-explanation homological-algebra






      share|cite|improve this question















      share|cite|improve this question













      share|cite|improve this question




      share|cite|improve this question








      edited Dec 29 '18 at 16:03







      idriskameni

















      asked Dec 29 '18 at 15:56









      idriskameniidriskameni

      732321




      732321






















          2 Answers
          2






          active

          oldest

          votes


















          3












          $begingroup$

          One of the things one learns about the $mathrm{Ext}$ functor is that it is 'balanced', in the sense that to compute $mathrm{Ext}^*(M,N)$ for two left $R$-modules $M$ and $N$, one can either resolve $M$ by projectives $P_*to M$ or $N$ by injectives $Nto I^*$. What one then shows is that the two morphisms of chain complexes $hom(P_*,N)longleftarrow hom(P_*,I^*) longrightarrow hom(M,I^*)$ obtained by pre- or post-composition with the augmentation maps for $M$ and $N$ are quasi-isomorphisms, so this shows the definitions agree.



          Since localization is exact, it commutes with homology. A general fact of life is exact functors commute with homology. In this case, if $C$ is a complex of $R$-modules, then we have a map of chain complexes $S^{-1}H(C) to H(S^{-1}C)$ that sends $s^{-1}[z]$ to $[s^{-1}z]$, and this is an isomorphism. This follos from the fact that if $F$ is flat then $F otimes H(C) to H(Fotimes C)$ is an isomorphism, for example by the Kunneth formula.



          The finite generation is required to commute localization and homs, that is, there is in general a map $S^{-1}hom_R(M,N) to hom_{S^{-1}R}(S^{-1}M,S^{-1}N)$ but this fails to be an isomorphism for $M$ and $N$ arbitrary. If $M$ is finitely presented, it is an isomorphism. In case $R$ is noetherian, this is the same as $M$ being finitely generated. Since Ext is Hom at 0, the general claim is false.



          Finally, you want that resolution because it computes the right hand side of your formula, i.e. the Ext over the localized ring with arguments the localized modules.






          share|cite|improve this answer











          $endgroup$





















            1












            $begingroup$


            1. The Ext defined using injectives is equivalent to the one defined using projectives, it is called balanced Ext.


            2. There is a natural map $S^{-1}operatorname{Hom}_R(M,N)to operatorname{Hom}_{S^{-1}R}(S^{-1}M,S^{-1}N)$ by $frac{f}{s}mapsto left[frac{x}{t}mapsto frac{f(x)}{st}right]$.
              Note that when $M$ is free of finite rank, the map is isomorphism (see 3.).
              When $M$ is finitely presented (if $R$ is noetherian, then any finitely generated modules are finitely presented), Let
              $$R^{m}to R^nto Mto 0$$
              Be right short exact sequence. Consider the following diagram
              $$begin{array}{ccccccc}
              0 & to &S^{-1}operatorname{Hom}_R(M,N)& to & S^{-1}operatorname{Hom}_R(R^n,N)&to& S^{-1}operatorname{Hom}_R(R^m,N)\
              &&downarrow && downarrow&& downarrow\
              0& to &operatorname{Hom}_{S^{-1}R}(S^{-1}M,S^{-1}N) & to & operatorname{Hom}_{S^{-1}R}(S^{-1}R^n,S^{-1}N)& to & operatorname{Hom}_{S^{-1}R}(S^{-1}R^{m},S^{-1}N)
              end{array}$$

              The right two columns of maps are isomorphic, thus so is the first one.


            3. The problem occurs when it involves infinite products. For example,
              $$operatorname{Hom}_{S^{-1}R}(S^{-1}R^{oplus Lambda},S^{-1}N)=(S^{-1}N)^{Lambda}qquad S^{-1}operatorname{Hom}_R(R^{oplus Lambda},N)=S^{-1}big(N^{ Lambda}big)$$
              They are quite different for infinite $Lambda$. For example, $R=mathbb{Z}=N$, and $S={1,2,4,8ldots }$ and $Lambda= mathbb{Z}_{geq 0}$, then there is no element of $S^{-1}(N^{Lambda})$ corresponding to
              $$left(1,frac{1}{2},frac{1}{4},frac{1}{8},ldots right)in (S^{-1}N)^{Lambda}$$


            4. We didn't, you need to check the proof again.







            share|cite|improve this answer









            $endgroup$









            • 1




              $begingroup$
              Note that your answer to 4. is not quite right.
              $endgroup$
              – Pedro Tamaroff
              Dec 29 '18 at 16:55











            Your Answer





            StackExchange.ifUsing("editor", function () {
            return StackExchange.using("mathjaxEditing", function () {
            StackExchange.MarkdownEditor.creationCallbacks.add(function (editor, postfix) {
            StackExchange.mathjaxEditing.prepareWmdForMathJax(editor, postfix, [["$", "$"], ["\\(","\\)"]]);
            });
            });
            }, "mathjax-editing");

            StackExchange.ready(function() {
            var channelOptions = {
            tags: "".split(" "),
            id: "69"
            };
            initTagRenderer("".split(" "), "".split(" "), channelOptions);

            StackExchange.using("externalEditor", function() {
            // Have to fire editor after snippets, if snippets enabled
            if (StackExchange.settings.snippets.snippetsEnabled) {
            StackExchange.using("snippets", function() {
            createEditor();
            });
            }
            else {
            createEditor();
            }
            });

            function createEditor() {
            StackExchange.prepareEditor({
            heartbeatType: 'answer',
            autoActivateHeartbeat: false,
            convertImagesToLinks: true,
            noModals: true,
            showLowRepImageUploadWarning: true,
            reputationToPostImages: 10,
            bindNavPrevention: true,
            postfix: "",
            imageUploader: {
            brandingHtml: "Powered by u003ca class="icon-imgur-white" href="https://imgur.com/"u003eu003c/au003e",
            contentPolicyHtml: "User contributions licensed under u003ca href="https://creativecommons.org/licenses/by-sa/3.0/"u003ecc by-sa 3.0 with attribution requiredu003c/au003e u003ca href="https://stackoverflow.com/legal/content-policy"u003e(content policy)u003c/au003e",
            allowUrls: true
            },
            noCode: true, onDemand: true,
            discardSelector: ".discard-answer"
            ,immediatelyShowMarkdownHelp:true
            });


            }
            });














            draft saved

            draft discarded


















            StackExchange.ready(
            function () {
            StackExchange.openid.initPostLogin('.new-post-login', 'https%3a%2f%2fmath.stackexchange.com%2fquestions%2f3055973%2fs-1extn-ra-b-cong-extn-s-1rs-1a-s-1b%23new-answer', 'question_page');
            }
            );

            Post as a guest















            Required, but never shown

























            2 Answers
            2






            active

            oldest

            votes








            2 Answers
            2






            active

            oldest

            votes









            active

            oldest

            votes






            active

            oldest

            votes









            3












            $begingroup$

            One of the things one learns about the $mathrm{Ext}$ functor is that it is 'balanced', in the sense that to compute $mathrm{Ext}^*(M,N)$ for two left $R$-modules $M$ and $N$, one can either resolve $M$ by projectives $P_*to M$ or $N$ by injectives $Nto I^*$. What one then shows is that the two morphisms of chain complexes $hom(P_*,N)longleftarrow hom(P_*,I^*) longrightarrow hom(M,I^*)$ obtained by pre- or post-composition with the augmentation maps for $M$ and $N$ are quasi-isomorphisms, so this shows the definitions agree.



            Since localization is exact, it commutes with homology. A general fact of life is exact functors commute with homology. In this case, if $C$ is a complex of $R$-modules, then we have a map of chain complexes $S^{-1}H(C) to H(S^{-1}C)$ that sends $s^{-1}[z]$ to $[s^{-1}z]$, and this is an isomorphism. This follos from the fact that if $F$ is flat then $F otimes H(C) to H(Fotimes C)$ is an isomorphism, for example by the Kunneth formula.



            The finite generation is required to commute localization and homs, that is, there is in general a map $S^{-1}hom_R(M,N) to hom_{S^{-1}R}(S^{-1}M,S^{-1}N)$ but this fails to be an isomorphism for $M$ and $N$ arbitrary. If $M$ is finitely presented, it is an isomorphism. In case $R$ is noetherian, this is the same as $M$ being finitely generated. Since Ext is Hom at 0, the general claim is false.



            Finally, you want that resolution because it computes the right hand side of your formula, i.e. the Ext over the localized ring with arguments the localized modules.






            share|cite|improve this answer











            $endgroup$


















              3












              $begingroup$

              One of the things one learns about the $mathrm{Ext}$ functor is that it is 'balanced', in the sense that to compute $mathrm{Ext}^*(M,N)$ for two left $R$-modules $M$ and $N$, one can either resolve $M$ by projectives $P_*to M$ or $N$ by injectives $Nto I^*$. What one then shows is that the two morphisms of chain complexes $hom(P_*,N)longleftarrow hom(P_*,I^*) longrightarrow hom(M,I^*)$ obtained by pre- or post-composition with the augmentation maps for $M$ and $N$ are quasi-isomorphisms, so this shows the definitions agree.



              Since localization is exact, it commutes with homology. A general fact of life is exact functors commute with homology. In this case, if $C$ is a complex of $R$-modules, then we have a map of chain complexes $S^{-1}H(C) to H(S^{-1}C)$ that sends $s^{-1}[z]$ to $[s^{-1}z]$, and this is an isomorphism. This follos from the fact that if $F$ is flat then $F otimes H(C) to H(Fotimes C)$ is an isomorphism, for example by the Kunneth formula.



              The finite generation is required to commute localization and homs, that is, there is in general a map $S^{-1}hom_R(M,N) to hom_{S^{-1}R}(S^{-1}M,S^{-1}N)$ but this fails to be an isomorphism for $M$ and $N$ arbitrary. If $M$ is finitely presented, it is an isomorphism. In case $R$ is noetherian, this is the same as $M$ being finitely generated. Since Ext is Hom at 0, the general claim is false.



              Finally, you want that resolution because it computes the right hand side of your formula, i.e. the Ext over the localized ring with arguments the localized modules.






              share|cite|improve this answer











              $endgroup$
















                3












                3








                3





                $begingroup$

                One of the things one learns about the $mathrm{Ext}$ functor is that it is 'balanced', in the sense that to compute $mathrm{Ext}^*(M,N)$ for two left $R$-modules $M$ and $N$, one can either resolve $M$ by projectives $P_*to M$ or $N$ by injectives $Nto I^*$. What one then shows is that the two morphisms of chain complexes $hom(P_*,N)longleftarrow hom(P_*,I^*) longrightarrow hom(M,I^*)$ obtained by pre- or post-composition with the augmentation maps for $M$ and $N$ are quasi-isomorphisms, so this shows the definitions agree.



                Since localization is exact, it commutes with homology. A general fact of life is exact functors commute with homology. In this case, if $C$ is a complex of $R$-modules, then we have a map of chain complexes $S^{-1}H(C) to H(S^{-1}C)$ that sends $s^{-1}[z]$ to $[s^{-1}z]$, and this is an isomorphism. This follos from the fact that if $F$ is flat then $F otimes H(C) to H(Fotimes C)$ is an isomorphism, for example by the Kunneth formula.



                The finite generation is required to commute localization and homs, that is, there is in general a map $S^{-1}hom_R(M,N) to hom_{S^{-1}R}(S^{-1}M,S^{-1}N)$ but this fails to be an isomorphism for $M$ and $N$ arbitrary. If $M$ is finitely presented, it is an isomorphism. In case $R$ is noetherian, this is the same as $M$ being finitely generated. Since Ext is Hom at 0, the general claim is false.



                Finally, you want that resolution because it computes the right hand side of your formula, i.e. the Ext over the localized ring with arguments the localized modules.






                share|cite|improve this answer











                $endgroup$



                One of the things one learns about the $mathrm{Ext}$ functor is that it is 'balanced', in the sense that to compute $mathrm{Ext}^*(M,N)$ for two left $R$-modules $M$ and $N$, one can either resolve $M$ by projectives $P_*to M$ or $N$ by injectives $Nto I^*$. What one then shows is that the two morphisms of chain complexes $hom(P_*,N)longleftarrow hom(P_*,I^*) longrightarrow hom(M,I^*)$ obtained by pre- or post-composition with the augmentation maps for $M$ and $N$ are quasi-isomorphisms, so this shows the definitions agree.



                Since localization is exact, it commutes with homology. A general fact of life is exact functors commute with homology. In this case, if $C$ is a complex of $R$-modules, then we have a map of chain complexes $S^{-1}H(C) to H(S^{-1}C)$ that sends $s^{-1}[z]$ to $[s^{-1}z]$, and this is an isomorphism. This follos from the fact that if $F$ is flat then $F otimes H(C) to H(Fotimes C)$ is an isomorphism, for example by the Kunneth formula.



                The finite generation is required to commute localization and homs, that is, there is in general a map $S^{-1}hom_R(M,N) to hom_{S^{-1}R}(S^{-1}M,S^{-1}N)$ but this fails to be an isomorphism for $M$ and $N$ arbitrary. If $M$ is finitely presented, it is an isomorphism. In case $R$ is noetherian, this is the same as $M$ being finitely generated. Since Ext is Hom at 0, the general claim is false.



                Finally, you want that resolution because it computes the right hand side of your formula, i.e. the Ext over the localized ring with arguments the localized modules.







                share|cite|improve this answer














                share|cite|improve this answer



                share|cite|improve this answer








                edited Dec 29 '18 at 16:44

























                answered Dec 29 '18 at 16:23









                Pedro TamaroffPedro Tamaroff

                97.4k10153297




                97.4k10153297























                    1












                    $begingroup$


                    1. The Ext defined using injectives is equivalent to the one defined using projectives, it is called balanced Ext.


                    2. There is a natural map $S^{-1}operatorname{Hom}_R(M,N)to operatorname{Hom}_{S^{-1}R}(S^{-1}M,S^{-1}N)$ by $frac{f}{s}mapsto left[frac{x}{t}mapsto frac{f(x)}{st}right]$.
                      Note that when $M$ is free of finite rank, the map is isomorphism (see 3.).
                      When $M$ is finitely presented (if $R$ is noetherian, then any finitely generated modules are finitely presented), Let
                      $$R^{m}to R^nto Mto 0$$
                      Be right short exact sequence. Consider the following diagram
                      $$begin{array}{ccccccc}
                      0 & to &S^{-1}operatorname{Hom}_R(M,N)& to & S^{-1}operatorname{Hom}_R(R^n,N)&to& S^{-1}operatorname{Hom}_R(R^m,N)\
                      &&downarrow && downarrow&& downarrow\
                      0& to &operatorname{Hom}_{S^{-1}R}(S^{-1}M,S^{-1}N) & to & operatorname{Hom}_{S^{-1}R}(S^{-1}R^n,S^{-1}N)& to & operatorname{Hom}_{S^{-1}R}(S^{-1}R^{m},S^{-1}N)
                      end{array}$$

                      The right two columns of maps are isomorphic, thus so is the first one.


                    3. The problem occurs when it involves infinite products. For example,
                      $$operatorname{Hom}_{S^{-1}R}(S^{-1}R^{oplus Lambda},S^{-1}N)=(S^{-1}N)^{Lambda}qquad S^{-1}operatorname{Hom}_R(R^{oplus Lambda},N)=S^{-1}big(N^{ Lambda}big)$$
                      They are quite different for infinite $Lambda$. For example, $R=mathbb{Z}=N$, and $S={1,2,4,8ldots }$ and $Lambda= mathbb{Z}_{geq 0}$, then there is no element of $S^{-1}(N^{Lambda})$ corresponding to
                      $$left(1,frac{1}{2},frac{1}{4},frac{1}{8},ldots right)in (S^{-1}N)^{Lambda}$$


                    4. We didn't, you need to check the proof again.







                    share|cite|improve this answer









                    $endgroup$









                    • 1




                      $begingroup$
                      Note that your answer to 4. is not quite right.
                      $endgroup$
                      – Pedro Tamaroff
                      Dec 29 '18 at 16:55
















                    1












                    $begingroup$


                    1. The Ext defined using injectives is equivalent to the one defined using projectives, it is called balanced Ext.


                    2. There is a natural map $S^{-1}operatorname{Hom}_R(M,N)to operatorname{Hom}_{S^{-1}R}(S^{-1}M,S^{-1}N)$ by $frac{f}{s}mapsto left[frac{x}{t}mapsto frac{f(x)}{st}right]$.
                      Note that when $M$ is free of finite rank, the map is isomorphism (see 3.).
                      When $M$ is finitely presented (if $R$ is noetherian, then any finitely generated modules are finitely presented), Let
                      $$R^{m}to R^nto Mto 0$$
                      Be right short exact sequence. Consider the following diagram
                      $$begin{array}{ccccccc}
                      0 & to &S^{-1}operatorname{Hom}_R(M,N)& to & S^{-1}operatorname{Hom}_R(R^n,N)&to& S^{-1}operatorname{Hom}_R(R^m,N)\
                      &&downarrow && downarrow&& downarrow\
                      0& to &operatorname{Hom}_{S^{-1}R}(S^{-1}M,S^{-1}N) & to & operatorname{Hom}_{S^{-1}R}(S^{-1}R^n,S^{-1}N)& to & operatorname{Hom}_{S^{-1}R}(S^{-1}R^{m},S^{-1}N)
                      end{array}$$

                      The right two columns of maps are isomorphic, thus so is the first one.


                    3. The problem occurs when it involves infinite products. For example,
                      $$operatorname{Hom}_{S^{-1}R}(S^{-1}R^{oplus Lambda},S^{-1}N)=(S^{-1}N)^{Lambda}qquad S^{-1}operatorname{Hom}_R(R^{oplus Lambda},N)=S^{-1}big(N^{ Lambda}big)$$
                      They are quite different for infinite $Lambda$. For example, $R=mathbb{Z}=N$, and $S={1,2,4,8ldots }$ and $Lambda= mathbb{Z}_{geq 0}$, then there is no element of $S^{-1}(N^{Lambda})$ corresponding to
                      $$left(1,frac{1}{2},frac{1}{4},frac{1}{8},ldots right)in (S^{-1}N)^{Lambda}$$


                    4. We didn't, you need to check the proof again.







                    share|cite|improve this answer









                    $endgroup$









                    • 1




                      $begingroup$
                      Note that your answer to 4. is not quite right.
                      $endgroup$
                      – Pedro Tamaroff
                      Dec 29 '18 at 16:55














                    1












                    1








                    1





                    $begingroup$


                    1. The Ext defined using injectives is equivalent to the one defined using projectives, it is called balanced Ext.


                    2. There is a natural map $S^{-1}operatorname{Hom}_R(M,N)to operatorname{Hom}_{S^{-1}R}(S^{-1}M,S^{-1}N)$ by $frac{f}{s}mapsto left[frac{x}{t}mapsto frac{f(x)}{st}right]$.
                      Note that when $M$ is free of finite rank, the map is isomorphism (see 3.).
                      When $M$ is finitely presented (if $R$ is noetherian, then any finitely generated modules are finitely presented), Let
                      $$R^{m}to R^nto Mto 0$$
                      Be right short exact sequence. Consider the following diagram
                      $$begin{array}{ccccccc}
                      0 & to &S^{-1}operatorname{Hom}_R(M,N)& to & S^{-1}operatorname{Hom}_R(R^n,N)&to& S^{-1}operatorname{Hom}_R(R^m,N)\
                      &&downarrow && downarrow&& downarrow\
                      0& to &operatorname{Hom}_{S^{-1}R}(S^{-1}M,S^{-1}N) & to & operatorname{Hom}_{S^{-1}R}(S^{-1}R^n,S^{-1}N)& to & operatorname{Hom}_{S^{-1}R}(S^{-1}R^{m},S^{-1}N)
                      end{array}$$

                      The right two columns of maps are isomorphic, thus so is the first one.


                    3. The problem occurs when it involves infinite products. For example,
                      $$operatorname{Hom}_{S^{-1}R}(S^{-1}R^{oplus Lambda},S^{-1}N)=(S^{-1}N)^{Lambda}qquad S^{-1}operatorname{Hom}_R(R^{oplus Lambda},N)=S^{-1}big(N^{ Lambda}big)$$
                      They are quite different for infinite $Lambda$. For example, $R=mathbb{Z}=N$, and $S={1,2,4,8ldots }$ and $Lambda= mathbb{Z}_{geq 0}$, then there is no element of $S^{-1}(N^{Lambda})$ corresponding to
                      $$left(1,frac{1}{2},frac{1}{4},frac{1}{8},ldots right)in (S^{-1}N)^{Lambda}$$


                    4. We didn't, you need to check the proof again.







                    share|cite|improve this answer









                    $endgroup$




                    1. The Ext defined using injectives is equivalent to the one defined using projectives, it is called balanced Ext.


                    2. There is a natural map $S^{-1}operatorname{Hom}_R(M,N)to operatorname{Hom}_{S^{-1}R}(S^{-1}M,S^{-1}N)$ by $frac{f}{s}mapsto left[frac{x}{t}mapsto frac{f(x)}{st}right]$.
                      Note that when $M$ is free of finite rank, the map is isomorphism (see 3.).
                      When $M$ is finitely presented (if $R$ is noetherian, then any finitely generated modules are finitely presented), Let
                      $$R^{m}to R^nto Mto 0$$
                      Be right short exact sequence. Consider the following diagram
                      $$begin{array}{ccccccc}
                      0 & to &S^{-1}operatorname{Hom}_R(M,N)& to & S^{-1}operatorname{Hom}_R(R^n,N)&to& S^{-1}operatorname{Hom}_R(R^m,N)\
                      &&downarrow && downarrow&& downarrow\
                      0& to &operatorname{Hom}_{S^{-1}R}(S^{-1}M,S^{-1}N) & to & operatorname{Hom}_{S^{-1}R}(S^{-1}R^n,S^{-1}N)& to & operatorname{Hom}_{S^{-1}R}(S^{-1}R^{m},S^{-1}N)
                      end{array}$$

                      The right two columns of maps are isomorphic, thus so is the first one.


                    3. The problem occurs when it involves infinite products. For example,
                      $$operatorname{Hom}_{S^{-1}R}(S^{-1}R^{oplus Lambda},S^{-1}N)=(S^{-1}N)^{Lambda}qquad S^{-1}operatorname{Hom}_R(R^{oplus Lambda},N)=S^{-1}big(N^{ Lambda}big)$$
                      They are quite different for infinite $Lambda$. For example, $R=mathbb{Z}=N$, and $S={1,2,4,8ldots }$ and $Lambda= mathbb{Z}_{geq 0}$, then there is no element of $S^{-1}(N^{Lambda})$ corresponding to
                      $$left(1,frac{1}{2},frac{1}{4},frac{1}{8},ldots right)in (S^{-1}N)^{Lambda}$$


                    4. We didn't, you need to check the proof again.








                    share|cite|improve this answer












                    share|cite|improve this answer



                    share|cite|improve this answer










                    answered Dec 29 '18 at 16:51









                    Cube BearCube Bear

                    622212




                    622212








                    • 1




                      $begingroup$
                      Note that your answer to 4. is not quite right.
                      $endgroup$
                      – Pedro Tamaroff
                      Dec 29 '18 at 16:55














                    • 1




                      $begingroup$
                      Note that your answer to 4. is not quite right.
                      $endgroup$
                      – Pedro Tamaroff
                      Dec 29 '18 at 16:55








                    1




                    1




                    $begingroup$
                    Note that your answer to 4. is not quite right.
                    $endgroup$
                    – Pedro Tamaroff
                    Dec 29 '18 at 16:55




                    $begingroup$
                    Note that your answer to 4. is not quite right.
                    $endgroup$
                    – Pedro Tamaroff
                    Dec 29 '18 at 16:55


















                    draft saved

                    draft discarded




















































                    Thanks for contributing an answer to Mathematics Stack Exchange!


                    • Please be sure to answer the question. Provide details and share your research!

                    But avoid



                    • Asking for help, clarification, or responding to other answers.

                    • Making statements based on opinion; back them up with references or personal experience.


                    Use MathJax to format equations. MathJax reference.


                    To learn more, see our tips on writing great answers.




                    draft saved


                    draft discarded














                    StackExchange.ready(
                    function () {
                    StackExchange.openid.initPostLogin('.new-post-login', 'https%3a%2f%2fmath.stackexchange.com%2fquestions%2f3055973%2fs-1extn-ra-b-cong-extn-s-1rs-1a-s-1b%23new-answer', 'question_page');
                    }
                    );

                    Post as a guest















                    Required, but never shown





















































                    Required, but never shown














                    Required, but never shown












                    Required, but never shown







                    Required, but never shown

































                    Required, but never shown














                    Required, but never shown












                    Required, but never shown







                    Required, but never shown







                    Popular posts from this blog

                    How do I know what Microsoft account the skydrive app is syncing to?

                    Grease: Live!

                    When does type information flow backwards in C++?